{ "cells": [ { "cell_type": "markdown", "metadata": {}, "source": [ "# Exercise 12.3 - Solution\n", "## Discriminative localization (CAM)\n", "In this exercise we will create class activation maps (CAMs) for predictions made by a model trained to classify magentic phases (see [Exercise 7_1](Exercise_7_1_solution.ipynb)).\n", " 1. Pick out two correctly and two wrongly classified images classified with a convolutional network.\n", " 2. Look at Exercise 8.1 to extract weights and feature maps from the trained network model.\n", " 3. Create and plot the class activation maps and compare them with the images in order to see which regions lead to the prediction.\n" ] }, { "cell_type": "code", "execution_count": 3, "metadata": {}, "outputs": [ { "name": "stdout", "output_type": "stream", "text": [ "keras version 2.4.0\n" ] } ], "source": [ "from tensorflow import keras\n", "import numpy as np\n", "callbacks = keras.callbacks\n", "layers = keras.layers\n", "\n" ] }, { "cell_type": "markdown", "metadata": {}, "source": [ "### Load and prepare dataset\n", "See https://doi.org/10.1038/nphys4035 for more information" ] }, { "cell_type": "code", "execution_count": 4, "metadata": {}, "outputs": [], "source": [ "import gdown\n", "url = \"https://drive.google.com/u/0/uc?export=download&confirm=HgGH&id=1Ihxt1hb3Kyv0IrjHlsYb9x9QY7l7n2Sl\"\n", "output = 'ising_data.npz'\n", "gdown.download(url, output, quiet=True)\n", "\n", "f = np.load(output)\n", "n_train = 20000\n", "\n", "x_train, x_test = f[\"C\"][:n_train], f[\"C\"][n_train:]\n", "T_train, T_test = f[\"T\"][:n_train], f[\"T\"][n_train:]\n", "\n", "Tc = 2.27\n", "y_train = np.zeros_like(T_train)\n", "y_train[T_train > Tc] = 1\n", "y_train = keras.utils.to_categorical(y_train, 2)\n", "\n", "y_test = np.zeros_like(T_test)\n", "y_test[T_test > Tc] = 1\n", "y_test = keras.utils.to_categorical(y_test, 2)" ] }, { "cell_type": "markdown", "metadata": {}, "source": [ "### Plot data" ] }, { "cell_type": "code", "execution_count": 5, "metadata": {}, "outputs": [ { "data": { "image/png": "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\n", "text/plain": [ "
" ] }, "metadata": { "needs_background": "light" }, "output_type": "display_data" } ], "source": [ "import matplotlib.pyplot as plt\n", "\n", "for i,j in enumerate(np.random.choice(n_train, 6)):\n", " plt.subplot(2,3,i+1)\n", " image = x_train[j]\n", " plot = plt.imshow(image)\n", " plt.title(\"T: %.2f\" % T_train[j])\n", "\n", "plt.tight_layout()\n", "plt.show()" ] }, { "cell_type": "markdown", "metadata": {}, "source": [ "### Definition of the model\n", "\n", "Define a CNN for discriminative localization. Note that the CNN must use `GlobalAveragePooling2D` after the convolutional part and must not feature more than a single fully-connected layer as output." ] }, { "cell_type": "code", "execution_count": 6, "metadata": {}, "outputs": [ { "name": "stdout", "output_type": "stream", "text": [ "Model: \"sequential\"\n", "_________________________________________________________________\n", "Layer (type) Output Shape Param # \n", "=================================================================\n", "reshape (Reshape) (None, 32, 32, 1) 0 \n", "_________________________________________________________________\n", "conv2d (Conv2D) (None, 32, 32, 16) 160 \n", "_________________________________________________________________\n", "conv2d_1 (Conv2D) (None, 32, 32, 16) 2320 \n", "_________________________________________________________________\n", "max_pooling2d (MaxPooling2D) (None, 16, 16, 16) 0 \n", "_________________________________________________________________\n", "conv2d_2 (Conv2D) (None, 16, 16, 32) 4640 \n", "_________________________________________________________________\n", "conv2d_3 (Conv2D) (None, 16, 16, 32) 9248 \n", "_________________________________________________________________\n", "global_average_pooling2d (Gl (None, 32) 0 \n", "_________________________________________________________________\n", "dropout (Dropout) (None, 32) 0 \n", "_________________________________________________________________\n", "dense (Dense) (None, 2) 66 \n", "=================================================================\n", "Total params: 16,434\n", "Trainable params: 16,434\n", "Non-trainable params: 0\n", "_________________________________________________________________\n" ] } ], "source": [ "model = keras.models.Sequential()\n", "model.add(layers.InputLayer(input_shape=(32, 32)))\n", "model.add(layers.Reshape((32, 32,1)))\n", "model.add(layers.Convolution2D(16, (3, 3), padding='same', activation='relu'))\n", "model.add(layers.Convolution2D(16, (3, 3), padding='same', activation='relu'))\n", "model.add(layers.MaxPooling2D((2, 2)))\n", "model.add(layers.Convolution2D(32, (3, 3), padding='same', activation='relu'))\n", "model.add(layers.Convolution2D(32, (3, 3), padding='same', activation='relu'))\n", "model.add(layers.GlobalAveragePooling2D())\n", "model.add(layers.Dropout(0.25))\n", "model.add(layers.Dense(2, activation='softmax'))\n", "\n", "model.summary()" ] }, { "cell_type": "markdown", "metadata": {}, "source": [ "### prepare model for training" ] }, { "cell_type": "code", "execution_count": 7, "metadata": {}, "outputs": [], "source": [ "model.compile(\n", " loss='binary_crossentropy',\n", " optimizer=keras.optimizers.Adam(0.001),\n", " metrics=['accuracy'])\n" ] }, { "cell_type": "code", "execution_count": 8, "metadata": {}, "outputs": [ { "name": "stdout", "output_type": "stream", "text": [ "Epoch 1/50\n", "282/282 - 26s - loss: 0.1005 - accuracy: 0.9548 - val_loss: 0.0427 - val_accuracy: 0.9815\n", "Epoch 2/50\n", "282/282 - 24s - loss: 0.0473 - accuracy: 0.9801 - val_loss: 0.0367 - val_accuracy: 0.9855\n", "Epoch 3/50\n", "282/282 - 24s - loss: 0.0448 - accuracy: 0.9816 - val_loss: 0.0361 - val_accuracy: 0.9835\n", "Epoch 4/50\n", "282/282 - 25s - loss: 0.0453 - accuracy: 0.9813 - val_loss: 0.0359 - val_accuracy: 0.9855\n", "Epoch 5/50\n", "282/282 - 24s - loss: 0.0454 - accuracy: 0.9808 - val_loss: 0.0368 - val_accuracy: 0.9835\n", "Epoch 6/50\n", "282/282 - 24s - loss: 0.0457 - accuracy: 0.9820 - val_loss: 0.0368 - val_accuracy: 0.9855\n", "\n", "Epoch 00006: ReduceLROnPlateau reducing learning rate to 0.0006700000318232924.\n", "Epoch 7/50\n", "282/282 - 25s - loss: 0.0444 - accuracy: 0.9821 - val_loss: 0.0370 - val_accuracy: 0.9840\n", "Epoch 8/50\n", "282/282 - 14s - loss: 0.0426 - accuracy: 0.9822 - val_loss: 0.0382 - val_accuracy: 0.9830\n", "\n", "Epoch 00008: ReduceLROnPlateau reducing learning rate to 0.0004489000252215192.\n", "Epoch 9/50\n", "282/282 - 13s - loss: 0.0422 - accuracy: 0.9826 - val_loss: 0.0364 - val_accuracy: 0.9860\n", "Epoch 00009: early stopping\n" ] } ], "source": [ "results = model.fit(x_train, y_train,\n", " batch_size=64,\n", " epochs=50,\n", " verbose=2,\n", " validation_split=0.1,\n", " callbacks=[\n", " callbacks.EarlyStopping(patience=5, verbose=1),\n", " callbacks.ReduceLROnPlateau(factor=0.67, patience=2, verbose=1)]\n", " )" ] }, { "cell_type": "markdown", "metadata": {}, "source": [ "### Evaluate training" ] }, { "cell_type": "code", "execution_count": 9, "metadata": {}, "outputs": [ { "data": { "text/plain": [ "" ] }, "execution_count": 9, "metadata": {}, "output_type": "execute_result" }, { "data": { "image/png": "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\n", "text/plain": [ "
" ] }, "metadata": { "needs_background": "light" }, "output_type": "display_data" } ], "source": [ "plt.figure(1, (12, 4))\n", "plt.subplot(1, 2, 1)\n", "plt.plot(results.history['loss'])\n", "plt.plot(results.history['val_loss'])\n", "plt.ylabel('loss')\n", "plt.xlabel('epoch')\n", "plt.legend(['train', 'val'], loc='upper right')" ] }, { "cell_type": "code", "execution_count": 10, "metadata": {}, "outputs": [ { "data": { "text/plain": [ "" ] }, "execution_count": 10, "metadata": {}, "output_type": "execute_result" }, { "data": { "image/png": "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\n", "text/plain": [ "
" ] }, "metadata": { "needs_background": "light" }, "output_type": "display_data" } ], "source": [ "plt.figure(1, (12, 4))\n", "plt.subplot(1, 2, 1)\n", "plt.plot(results.history['accuracy'])\n", "plt.plot(results.history['val_accuracy'])\n", "plt.ylabel('accuracy')\n", "plt.xlabel('epoch')\n", "plt.legend(['train', 'val'], loc='upper right')" ] }, { "cell_type": "markdown", "metadata": {}, "source": [ "## Create class activation maps" ] }, { "cell_type": "markdown", "metadata": {}, "source": [ "Extract the activations of the last convolutional layer." ] }, { "cell_type": "code", "execution_count": 11, "metadata": {}, "outputs": [ { "name": "stdout", "output_type": "stream", "text": [ "(6000, 16, 16, 32)\n" ] } ], "source": [ "conv = model.layers[-4]\n", "conv_func = keras.models.Model(model.inputs, conv.output)\n", "A = conv_func.predict(x_test)\n", "print(A.shape)" ] }, { "cell_type": "markdown", "metadata": {}, "source": [ "Create the class activation maps by omitting the global average pooling operation and applying the weights of the single classification layer to the extracted activations." ] }, { "cell_type": "code", "execution_count": 12, "metadata": {}, "outputs": [], "source": [ "W, b = model.layers[-1].get_weights()\n", "M = np.einsum('ixyz,zc->ixyc', A, W) + b" ] }, { "cell_type": "code", "execution_count": 13, "metadata": {}, "outputs": [ { "name": "stdout", "output_type": "stream", "text": [ "188/188 [==============================] - 1s 6ms/step\n" ] } ], "source": [ "y_pred = model.predict(x_test, verbose=1)\n", "\n", "y_pred = np.argmax(y_pred, axis=-1).round()\n", "y_true = np.argmax(y_test, axis=-1).round()" ] }, { "cell_type": "code", "execution_count": 14, "metadata": {}, "outputs": [], "source": [ "idx_correct = np.where(y_true == y_pred)[0]" ] }, { "cell_type": "code", "execution_count": 15, "metadata": {}, "outputs": [], "source": [ "idx_wrong = np.where(y_true != y_pred)[0]" ] }, { "cell_type": "markdown", "metadata": {}, "source": [ "### Plot the class activation maps for examples just below and above the critical temperature" ] }, { "cell_type": "code", "execution_count": 16, "metadata": {}, "outputs": [], "source": [ "def plot_CAM(M, x_test, idx):\n", " X = x_test[idx,...] # the image itself\n", " M1 = M[idx,..., 0] # activation map for T < Tc\n", " M2 = M[idx,..., 1] # activation map for T > Tc\n", "\n", " fig, (ax1, ax2) = plt.subplots(1, 2, sharey=True, figsize=(10, 4))\n", " fig.subplots_adjust(right=0.8, wspace=0.05)\n", "\n", " # plot image (twice)\n", " ax1.imshow(X, cmap=plt.cm.gray, extent=(0, 32, 0, 32))\n", " ax2.imshow(X, cmap=plt.cm.gray, extent=(0, 32, 0, 32))\n", "\n", " # plot overlay of class activations, perform upsampling by bilinear interpolation\n", " kwargs = dict(cmap=plt.cm.magma, extent=(0, 32, 0, 32),\n", " interpolation='bilinear', alpha=0.7)\n", " _1 = ax1.imshow(M1, **kwargs)\n", " _2 = ax2.imshow(M2, **kwargs)\n", "\n", " cbar = fig.colorbar(_1, cax=fig.add_axes([0.82, 0.1, 0.02, 0.8]))\n", " cbar.set_label('Class activation')\n", " ax1.set_title('Ferromagnetic CAM (T < Tc)')\n", " ax2.set_title('Paramagnetic CAM (T > Tc)')\n", " plt.show()" ] }, { "cell_type": "markdown", "metadata": {}, "source": [ "Plot correctly classified sample " ] }, { "cell_type": "code", "execution_count": 17, "metadata": {}, "outputs": [ { "data": { "image/png": "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\n", "text/plain": [ "
" ] }, "metadata": { "needs_background": "light" }, "output_type": "display_data" } ], "source": [ "plot_CAM(M, x_test, np.random.choice(idx_correct, 1)[0])" ] }, { "cell_type": "markdown", "metadata": {}, "source": [ "Plot wrongly classified sample " ] }, { "cell_type": "code", "execution_count": 18, "metadata": {}, "outputs": [ { "data": { "image/png": "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\n", "text/plain": [ "
" ] }, "metadata": { "needs_background": "light" }, "output_type": "display_data" } ], "source": [ "plot_CAM(M, x_test, np.random.choice(idx_wrong, 1)[0])" ] } ], "metadata": { "kernelspec": { "display_name": "Python 3", "language": "python", "name": "python3" }, "language_info": { "codemirror_mode": { "name": "ipython", "version": 3 }, "file_extension": ".py", "mimetype": "text/x-python", "name": "python", "nbconvert_exporter": "python", "pygments_lexer": "ipython3", "version": "3.6.9" } }, "nbformat": 4, "nbformat_minor": 4 }